A group of 6 students was asked, " How many hours did you watch television last week?" Here are their responses 17 , 15 , 5 , 8 , 20 Find the mean number of hours for these students. If necessary, round your answer to the nearest tenth.

Answers

Answer 1

Answer:

13 hours

Step-by-step explanation:

Mean=sum of observations/number of observations

=>17+15+5+8+20/5

=>65/5

=>13

Answer 2

Answer:  14.2

Step-by-step explanation:

The numbers are 17,15,5,8,20, and 20   and to find their mean you need to find their average which their sum divide by the number of values.

17 + 15 + 5 + 8 + 20 + 20 = 85

85 /6 = 14.1666...  and rounded to the nearest tenth is 14.2


Related Questions

This year, Zachary has been babysitting his young cousins after school for $70 a month. His uncle also
gave him an extra bonus of $100 for his excellent work. Since school started, Zachary has earned more
than $500. How many months ago did school start?

Answers

Answer:

8

Step-by-step explanation:

Simplify the expression: z8 * z-3

Answers

Answer:

z^5

Step-by-step explanation:

z^8  * z^-3

Since we are multiplying exponents and the bases are the same, we can add the exponents

z^(8-3)

z^5

27x + 45y and 7(9x + 7y) are these equivalent?

Answers

Answer:

No

Step-by-step explanation:

If you want to know whether they are equivalent or no, we will first have to find the value of both.

[tex]27x+45y[/tex]

Since there aren't any like terms here, your answer would be

[tex]27x+45y[/tex]

Let's solve for next one.

[tex]7(9x+7y)[/tex]

Let's add in parenthesis to 7.

[tex](7)(9x+7y)[/tex]

Now we will separate everything. And also you will have to add.

[tex](7)(9x)+(7)(7y)[/tex]

Now that we added we got our answer.

[tex]63x+49y[/tex]

Both values were not the same, so therefore, your answer is no, they are not equivalent.

Hope this helps!

No

Step-by-step explanation:

If you want to know whether they are equivalent or no, we will first have to find the value of both.

Since there aren't any like terms here, your answer would be

Let's solve for next one.

Let's add in parenthesis to 7.

Now we will separate everything. And also you will have to add.

Now that we added we got our answer.

Both values were not the same, so therefore, your answer is no, they are not equivalent.

Hope this helps!

Read more on Brainly.com - https://brainly.com/question/17379473#readmore

Jodi is considering taking online classes at two websites, LearnCenter and EduWorld. Each site requires that students pay a base membership fee, plus they charge a rate per class. The two graphs show the cost for taking online classes through the two websites. A graph titled Cost to Take Classes at LearnCenter has number of classes on the x-axis and cost in dollars on the y-axis. A line goes through points (0, 50) and (1, 70). A graph titled Cost to Take Classes at EduWorld has number of classes on the x-axis and cost in dollars on the y-axis. A line goes through points (0, 20) and (1, 60).

Answers

Answer:

B-LearnCenter charges a higher membership fee but a lower rate per class.

Step-by-step explanation:

Answer:B-learning for ever

Step-by-step explanation:

Please solve for a ! 24+5a=-a+6(5a-4)

Answers

Answer:

a = 2

Step-by-step explanation:

First of all you should remove parentheses by using distribution

24 + 5a = - a + 30a - 24

Then you should collect all like terms

24 + 5a = 29a - 24

After you should move the terms so the variables are only on one side.

5a - 29a = -24 - 24

Collect like terms

-24a = -48

After dividing both sides you get

a = 2

meaning of cube root

Answers

Answer:

The cube root of a number is a special value that, when used in a multiplication three times, gives that number. Example: 3 × 3 × 3 = 27, so the cube root of 27 is 3. See: Square Root. Cubes and Cube Roots.

What is the measurement of this angle

Answers

Answer:

Acute, 40 degree angle

Step-by-step explanation:

Take 90 - 50 and you get 40

I hope this helps you :)

-KeairaDickson

Answer:

40°

Step-by-step explanation:

Count how many 10s it takes to get from 90 degrees to 50 and you get your answer...

The next model of a sports car will cost 12.9% more than the current model. The current model costs $35,000. How much will the price increase in dollars

Answers

Answer:

The price increase in dollars= $4,515

Step-by-step explanation:

Cost of the next model = cost of the current model + 12.9% cost of the current model

Cost of the current model = $35,000

12.9% cost of the current model

= 12.9/100 × $35,000

= 0.129 × $35,000

= $4,515

Cost of the next model = cost of the current model + 12.9% cost of the current model

= $35,000 + $4,515

= $39,515

Cost of the next model = $39,515

The price increase in dollars= next model price - current model price

=$39,515 - $35,000

= $4,515

The price increase in dollars = $4,515

help please will mark brainliest

Answers

Answer:

i can't see that

Step-by-step explanation:

plzz sent me a clear photo

Answer:

1. 1

2. 1  ⋅  10 ^24

3. 1000000000

4. 1

5. 1  ⋅  10 ^20

6. 1  ⋅  10 ^30

7. 1  ⋅  10 ^27

8. 10000

9. 0.01

10. 0.0001

11. 0.1

Hopefully these are right, I simplified all of the problems...sorry if they are wrong

Step-by-step explanation:

You borrow $350 from your aunt and agree to repay her $400 ($350 principal + $50 interest) in 18 months. What interest rate (using simple interest, and to the nearest tenth) are you paying?

Answers

Answer:

The interest rate is 0.79% per month.

Step-by-step explanation:

Present value of borrowed amount = $350

Future value of borrowed amount = $400

Interest amount = $50

Time duration = 18 months or 1.5 years

Now we have to find the rate of interest by using the above information. Here below is the calculation of interest rate.

Interest rate =  P×R×T

50 = 350 × R×18

R = 0.007936

Or, R = 0.79% per month.

how do i solve this? xw+vy-xy-vw​

Answers

Here you go hopes this helps

Consider the composite function g (f (x)) = x StartRoot 6 EndRoot. If f(x) = 3x2, what is g(x)?
g (x) = StartRoot 2 x EndRoot
g (x) = StartRoot x + 3 EndRoot
g (x) = StartRoot 6 x EndRoot
g (x) = StartRoot 9 minus x EndRoot

Answers

Answer:

The answer is A on edge

Step-by-step explanation:

The required function g(x) = √2x

Given that,
The composite function g (f (x))= x √6
f(x) = 3x²
g(x) = To be determine.

What are functions?

Functions are the relationship between sets of values. e g y=f(x), for every value of x there is its exists in a set of y. x is the independent variable while Y is the dependent variable.

Here,
g (f (x))= x √6
g (3x²)= x √6
g (3x²)= √6x²
g (3x²)= √2*3x²
Let
x ⇆ 3x²
g (x)= √2x

Thus, the required function g(x) = √2x.

Learn more about function here:

brainly.com/question/21145944

#SPJ2

One year, Mr James travels 8 x 10^3 miles for his job. The next year he traveled 1 x 10^4 miles. How many more miles did he travel the second year than he did the first year? Please give me the answer and explain ToT

Answers

Answer:

2 000 miles

Step-by-step explanation:

In the first year, Mr James travels 8 x [tex]10^{3}[/tex] miles = 8 000 miles.

The second year, he traveled 1 x [tex]10^{4}[/tex] miles = 10 000 miles.

Comparing the distance covered in the two years,

10 000 miles - 8 000 miles = 2 000 miles

Therefore, Mr James travels 2 000 more miles in the second year than he did the first year.

Let's assume that Mr James conveys himself to his place of work all through the first year in his private car. But in the second year, there were some days in which the car was not available. So he had to go for his job by public transportation. This itch during those days, could cause an increase in the distance covered from his home to his place of work for the second year.

if f(x)=4-3x*2 then f(-2) is

Answers

Answer:

16

Step-by-step explanation:

f(-2)=4-3(-2)*2

f(-2)=4-3(-4)

f(-2)=4+12

f(-2)=16

The following data points represent the volume of gas in each race car driver's tank (in liters). Sort the data from least to greatest. Find the interquartile range (IQR) of the data set. \text{L}Lstart text, L, end text \text{L}Lstart text, L, end text

Answers

Answer:

its \text{L}Lstart text, L, end text

Step-by-step explanation:

The sorting of the dataset and the information from the five number summary, which includes the three quartiles and the minimum and maximum values indicates that we get;

1. 2.8, 4, 4.3, 6, 7.5, 8.5, 9, 11.6, 12, 12.1

2. The interquartile range is; 7.745

What is the five number summary of interquartile range of a data set?

The five number summary includes the first, second and third quartiles, Q₁, Q₂, and Q₃, and maximum and minimum values of the dataset, which serves to describe the characteristics of the data.

here, we have,

1. The sorted data from least to greatest can be presented as follows;

2.8, 4, 4.3, 6, 7.5, 8.5, 9, 11.6, 12, 12.1

Therefore, the original data in the question is already sorted from least to the greatest

2. The interquartile range, IQR, is the the difference between the third quartile, Q₃, and the first quartiles, Q₁, of the data.

IQR = Q₃ - Q₁

The dataset indicates that we get;

The number of datapoints in the dataset, n = 10

The first quartile = The (N + 1)/4 percentile value

First quartile, Q₁ = (10 + 1)/4 × 10 = The 2.75th value

The 2.75th value is the value 0.75 = 3/4 larger than the 2nd value, which can be found as follows;

Q₁ = The 2.75th value = 4 + (4.3 - 4) × (3/4)  = 4.225

The third quartile, Q₃ = The (N + 1) × (3/40th value, therefore;

Q₃ = The (10 + 1) × (3/4)th value = The 8.25th value (The value 0.25th more than the 8th value )

Q₃ = 11.6 + (12 - 11.6) × 0.25 = 11.7

The interquartile range, IQR = 11.7 - 4.225 = 7.475

The interquartile range = 7.475

Learn more on the interquartile range of a dataset here: brainly.com/question/13119591

#SPJ7

Complete question:

The following data points represent the volume of gas in each race car driver's tank (in liters).

Answer 2 questions about the data set.

1. Sort the data from least to greatest.

2.8

4

4.3

6

7.5

8.5

9

11.6

12

12.1

2. Find the interquartile range (IQR) of the data set.

L

I REALLY NEED HELP PLEASE HELP ME

Answers

Answer:

52^8

Step-by-step explanation:

26^8 = 208827064576 + 26

The area of a sector is 30 m2 in a circle with radius 4 m. What is the arc length of the sector?

Answers

Answer:

15 m

Step-by-step explanation:

The area of the circle is

A =pi r^2

A = pi 4^2 = 16 pi

The area of the sector is 30

The fraction is

30/16 pi  

Take this time 2pi which are the  radians of a circle

30 /16 pi * 2 pi = 15/4

This is the number of radians the angle is

The arc length is s = r * theta where theta is in radians

s = r  theta

  = 4 * 15/4

  = 15

Use the table to complete the work to find the missing value. Conversion Chart Pints Ounces 3 48 7 ? 30 480 3 pints 48 ounces 11 pints ? ounces How many ounces are in 11 pints? o 144 o160 o 176 o 192​

Answers

Answer:

176 ounces (C)

Step-by-step explanation:

if you divide 48 by 3 it will give you 16 and there are 11 pints so u wanna multiply 16 and 11 to 176 ounces..

also i got it right on edge :P

Answer: 176

Step-by-step explanation:

Ronald wants to paint his drawing room wall which measures 8 feet by 9 feet. The cost of the paint is $14 per square foot. How much will it cost Ronald to paint the wall

Answers

Answer:

$1036

Step-by-step explanation:

His room is 8 x 9 = 72 feet.  The cost of each square foot is $14.  

So 72ft x $14 = $1036.  So its going to cost him $1036.

i hope i helped :)

Answer: Answer is 1,008

Step-by-step explanation:

Because 9 x 8 is 72 ft

Take 72 + 14 is equal to 1,008.

What is the solution to this equation?
X-9 = 17
O A. x = 28
O B. x = 12
O c. x = 8
O D. x = 26

Answers

Answer:

Step-by-step explanation:

x -  9 = 17

x = 26

the answer is D

i need help its urgent its due today geometry

Answers

Rays, vertex, the vertex of the angle,BXC

What is the smallest positive number that is prime and 10 less than a perfect square?

Answers

Answer:

The problem states that the answer cannot be a perfect square or have prime factors less than $50$. Therefore, the answer will be the product of at least two different primes greater than $50$. The two smallest primes greater than $50$ are $53$ and $59$. Multiplying these two primes, we obtain the number $3127$, which is also the smallest number on the list of answer choices. So we are done, and the answer is $\boxed{\textbf{(A)}\ 3127}$.

Step-by-step explanation:

Answer:

71

Step-by-step explanation:

I just did the AOPS question, you can see the attachment down below.

Hope this helped! :)


Need help on 3 thank you

Answers

Answer:

Cost: $10.00

Number of Tickets:26

Step-by-step explanation:

Each ticket is $0.50

20*$0.50=$10.00

For the tickets:

$13.00/$0.50=26 tickets

Christine wants to buy strawberries and raspberries to bring to a party. Strawberries cost $1.65 per pound and raspberries cost $2.25 per pound. She only has $15 to spend on berries. Which inequality represents the situation where she buys x pounds of strawberries and y pounds of raspberries?

Answers

Answer:

1.65x + 2.25y ≤ 15

Step-by-step explanation:

Strawberries = $1.65 per pound Raspberries = $2.25 per pound

Amount with Christine = $15

Let

pounds of strawberries = x

pounds of raspberries = y

PxQx + PyQy ≤ 15

1.65x + 2.25y ≤ 15

The inequality which represent the situation where she buys x pounds of strawberries and y pounds of raspberries is 1.65x + 2.25y ≤ 15

1.65x + 2.25y less than or equal to 15

A scientist has two kinds of solutions. The first solution is Solution A, which is an $80\%$ acid solution. The second solution is Solution B, which is a $30\%$ acid solution. (a) Find the amount of Solution A (in mL) that must be added to $500$ mL of Solution B in order to produce a $70\%$ acid solution. (b) Find the amount of Solution A and Solution B (in mL) that can be combined in order to form a $100$ mL solution that is $50\%$ acid. (c) Does there exist a combination of Solution A and Solution B that is $90\%$ acid?

Answers

Answer:

Step-by-step explanation:

a ) Let the volume required be v .

v x .8 + 500 x .3 / ( v + 500 ) = .7

.8 v + 150 = .7 v + 350

.1 v = 200

v = 2000 mL

b )

Let the volume of solution A  required be v . The volume of solution of B required will be 100 - v

v x .8 + ( 100 - v ) x .3 / 100 = .5

.8 v + 30 - .3 v = 50

.5 v = 20

v = 40 mL

volume of solution B = 60 mL .

c )

It is not possible because 90 % is more than 80 % the concentration of solution A . Any mixture will have concentration less than 80 % .

volume of solution B = 60 mL

Let n be a whole number, and consider the statements below. p: n is a multiple of two. q: n is an even number. Which of the following is equivalent to ~q → ~p? A. ~q → ~p B. q → p C. p → q D. ~p →~

Answers

Answer:

C. p -> q

Step-by-step explanation:

Just did this on Edge2020. Hope this helps :)

PLZ HELP ME ! Solve for k

Answers

Step-by-step explanation

solve for x (2x+1) (x+29)​

Answers

The answer would be 2x^2+59x+29

Identify the error in each problem. Explain each error and provide the correction solution. Use at least two complete sentences. PLEASE SOMEONE HELP QUICK ITS DUE TOMORROW AND I WILL MARK YOU BRAINLIEST

Answers

Answer:

the mistake is the addition sign. When a number is next to a parenthesis it means multiplication

find 3+root 2/3-root 2=a+b root2

Answers

Answer:

a = [tex]\frac{11}{7}[/tex] ; b = [tex]\frac{6}{7}[/tex]

Step-by-step explanation:

[tex]\frac{3 + \sqrt{2}}{3 - \sqrt{2} } = a + b\sqrt{2} \\\\[/tex]

Rationalising [tex]\frac{3 + \sqrt{2}}{3 - \sqrt{2} }[/tex] gives :-

[tex]\frac{3 + \sqrt{2}}{3 - \sqrt{2} } = \frac{(3 + \sqrt{2})(3 + \sqrt{2})}{(3 - \sqrt{2})(3 + \sqrt{2}) } = \frac{(3 + \sqrt{2})^2}{3^2 - (\sqrt{2})^2 } = \frac{11 +6\sqrt{2} }{7}[/tex]

Comparing [tex]\frac{11 + 6\sqrt{2} }{7}[/tex] with [tex]a + b\sqrt{2}[/tex] gives

a = [tex]\frac{11}{7}[/tex] & b = [tex]\frac{6}{7}[/tex]

Answer:

[tex]\frac{11}{7}[/tex] + [tex]\frac{6}{7}[/tex] [tex]\sqrt{2}[/tex]

Step-by-step explanation:

Given

[tex]\frac{3+\sqrt{2} }{3-\sqrt{2} }[/tex]

Multiply the numerator/ denominator by the conjugate of the denominator.

The conjugate of 3 - [tex]\sqrt{2}[/tex] is 3 + [tex]\sqrt{2}[/tex] , thus

= [tex]\frac{(3+\sqrt{2})(3+\sqrt{2}) }{(3-\sqrt{2})(3+\sqrt{2}) }[/tex] ← expand numerator/ denominator using FOIL

= [tex]\frac{9+6\sqrt{2}+2 }{9-2}[/tex]

= [tex]\frac{11+6\sqrt{2} }{7}[/tex]

= [tex]\frac{11}{7}[/tex] + [tex]\frac{6}{7}[/tex] [tex]\sqrt{2}[/tex] ← in the form a + b[tex]\sqrt{2}[/tex]

with a = [tex]\frac{11}{7}[/tex] and b = [tex]\frac{6}{7}[/tex]

Other Questions
Solve the inequality. A. >> O B. x >7 C. x>-5 ) D. x if a cubic container has a side length of 50.0cm, what is the volume in liter? Complete the paragraph to explain how Angelina can notify readers that her report is just a draft. Angelina has decided to add the word DRAFT to the of the pages of her report as a watermark. To do this, she needs to navigate to the tab and then to the command group to click the Watermark icon and then the DRAFT option. If she wanted to include additional text, such as her initials, she could create a watermark. Which property is used in the problem below?2 (x + 4) = 2 x + 8the associative property the commutative propertythe distributive propertythe additive identity property which of the questions can be answered through science? what is the best water sport? why should we value honesty? why does water change into steam? what is the most important thing in life? Which explorer sailed the furthest NORTH?(2.1)O VespucciO PizarroO CabotO Columbus what is the domain and range of f(x)=2x+3 2. What is the name of the large island just south of Florida? A person is seeking a job in the lumber industry. Which Texas region would likely offer them the best opportunity for finding a job? Can yall help me? Answer the following Questions about Thank You Ma'am1. How does the author show us the type of person Mrs. Luella Bates Washington Jones is?2. How does the author show us the type of person the boy is?3. How does social pressure influence both of these characters? $36 for 4 pounds of shrimp as a unit rate The velocity of an object is given by the following function defined on a specified interval. Approximate the displacement of the object on this interval by subdividing the interval into the indicated number of subintervals. Use the left endpoint of each subinterval to compute the height of the rectangles. v = 1/(2t + 4) (m/s) for for 0 t 88; n = 22 Identify the pronoun and its type in the sentence below.Put those cookies on the dining room table.prounoun:type: IrepWhat is the solution of 4(2y + 1) = 2(y - 13)What is the answer Gwen releases a rock at rest from the top of a 40-m tower. If g = 9.8 m/s2 and air resistance is negligible, what is the speed of the rock as it hits the ground? G(x) = 2x^2 and h(x) = x^2+1 .What is (goh)^-1 and is it a function? I have 4 questions1. Suppose point T is between points R and V on a line. If RT = 63 units and RV = 131 units,then what is TV? 13119468802.Given point P is between M and N. If MN = 26, MP = x + 4, and PN = 2x + 1, what is the value of x? x = 3x = 7x = 12.5x = 223.Given M is the midpoint of HJ, HM = 4x - 12, and MJ = 3x + 9. What is the value of x? 4.If D is the midpoint of CE, DE = 2x + 4, and CE = 6x + 2, then what is CD? Jacki evaluated the expression below. 2 cubed (3 minus 1) + 4 (8 minus 12) = 2 cubed (2) + 4 (4) = 8 (2) + 16 = 16 + 16 = 32. What was Jackis error? Jacki should have simplified the exponent first. Jacki should have multiplied 4 and 8 first. Jacki did not subtract 12 from 8 correctly. Jacki should not have multiplied What number line correctly shows one way to find 2-6 ? Question 1 (1 point)When simplified [9(7 3) + 13] - [11 - (6 + 9)] equals:533943None of these